- Sat Jan 21, 2012 12:00 am
#72532
Complete Question Explanation
Weaken-SN. The correct answer choice is (B)
This relationship can be diagrammed as:
WRP = business wish(es) to retain power as long as possible
AR = act responsibly
WRP AR
Answer choice (A): Because this answer addresses government institutions, this cannot hurt the conclusion, which is about businesses. If anything, this may slightly support the argument. In the middle of the stimulus, the Speaker mentions that “The law’s application to human institutions certainly stands confirmed by history.” This answer affirms that statement by adding governments to the named list of human institutions.
Answer choice (B): This is the correct answer. If a public relations program can cause society to think an institution is socially responsible even when it is not, then an institution that wishes to retain power could act irresponsibly and then get a public relations firm to cover up the activities. In this way, the institution could wish to retain power but not act responsibly. Since this scenario allows the sufficient condition to occur without the necessary, this weakens the argument.
Answer choice (C): Many students hold this answer choice as a Contender. The answer is incorrect because the stimulus contemplates varying rates of power retention, especially between socially responsible and non-socially responsible institutions. If you read this answer thinking that the stimulus indicated socially responsible institutions do not lose power if socially responsible, then you made a quasi-Mistaken Reversal of the stimulus. There is never a presumption in the argument that power can be held indefinitely. If there were, this answer would be much more attractive.
Answer choice (D): The conclusion is clear in saying, “a business that wishes to retain power as long as it can...” The italicized phrase allows for the idea that businesses will eventually lose power and ultimately fail. Thus, this answer does not hurt the argument.
Answer choice (E): This is another attractive answer, and one that lured in many test takers. The answer states that even though some businesses acted responsibly (AR), they did not retain power (RP). If the difference between retaining power and wishing to retain power (WRP) is ignored, then this answer can be seen as attacking not the actual conclusion of the argument, but the Mistaken Reversal of the conclusion. As you learned from the discussion of answer choice (C) of the Carpet Market question, attacking the Mistaken Reversal of the conclusion does not hurt the actual conclusion. However, this answer is attractive because not only does it address elements of the conclusion, it also appears as the final answer choice. A test taker who did not
Weaken-SN. The correct answer choice is (B)
This relationship can be diagrammed as:
WRP = business wish(es) to retain power as long as possible
AR = act responsibly
WRP AR
Answer choice (A): Because this answer addresses government institutions, this cannot hurt the conclusion, which is about businesses. If anything, this may slightly support the argument. In the middle of the stimulus, the Speaker mentions that “The law’s application to human institutions certainly stands confirmed by history.” This answer affirms that statement by adding governments to the named list of human institutions.
Answer choice (B): This is the correct answer. If a public relations program can cause society to think an institution is socially responsible even when it is not, then an institution that wishes to retain power could act irresponsibly and then get a public relations firm to cover up the activities. In this way, the institution could wish to retain power but not act responsibly. Since this scenario allows the sufficient condition to occur without the necessary, this weakens the argument.
Answer choice (C): Many students hold this answer choice as a Contender. The answer is incorrect because the stimulus contemplates varying rates of power retention, especially between socially responsible and non-socially responsible institutions. If you read this answer thinking that the stimulus indicated socially responsible institutions do not lose power if socially responsible, then you made a quasi-Mistaken Reversal of the stimulus. There is never a presumption in the argument that power can be held indefinitely. If there were, this answer would be much more attractive.
Answer choice (D): The conclusion is clear in saying, “a business that wishes to retain power as long as it can...” The italicized phrase allows for the idea that businesses will eventually lose power and ultimately fail. Thus, this answer does not hurt the argument.
Answer choice (E): This is another attractive answer, and one that lured in many test takers. The answer states that even though some businesses acted responsibly (AR), they did not retain power (RP). If the difference between retaining power and wishing to retain power (WRP) is ignored, then this answer can be seen as attacking not the actual conclusion of the argument, but the Mistaken Reversal of the conclusion. As you learned from the discussion of answer choice (C) of the Carpet Market question, attacking the Mistaken Reversal of the conclusion does not hurt the actual conclusion. However, this answer is attractive because not only does it address elements of the conclusion, it also appears as the final answer choice. A test taker who did not